LSAT and Law School Admissions Forum

Get expert LSAT preparation and law school admissions advice from PowerScore Test Preparation.

 Toby
  • Posts: 33
  • Joined: Jun 05, 2017
|
#35826
Thank you so much, Francis!
 mrcheese
  • Posts: 32
  • Joined: Jun 27, 2018
|
#57802
I read the answer choice B as, "the area covered by >the library< in central Glenwood is approximately the same as that covered by central Redville.

Is the answer choice instead of speaking about how large the central areas of these two places are> If so, it would make more sense... but I did not understand it like that. I was thinking about library coverage.
 Claire Horan
PowerScore Staff
  • PowerScore Staff
  • Posts: 408
  • Joined: Apr 18, 2016
|
#60880
You've figured it out!

Answer choice (B) says "The area covered by central Glenwood is approximately the same size as that covered by central Redville." The sentence does not mention a library at all, so it is plainly referring to how large the central areas are. The reason that fact would strengthen the argument is that, if central Glenwood was a lot larger in area, people living in the area may still not live within walking distance of the new location.
 sparrrkk_
  • Posts: 26
  • Joined: Dec 13, 2019
|
#75386
Hi,

When I first read this stimulus, I noticed how the conclusion focuses on increasing the number of library users within walking distance to the library, while one of the premises say that there are many more people living in central Glenwood than in central Redville.
I thought it wasn't reasonable to infer something about the # of library users from the # of total people.
What if there are significantly more library users in central Redville?
If the library was moved to Glenwood where there are more people within walking distance to the library, that doesn't necessarily mean there are going to be more library users? What if people in Glenwood don't go to the library and buy books instead?

I understand why B is the correct answer, but was wondering if my reasoning makes sense? Would this also be a weakness of the argument in the stimulus?
 Adam Tyson
PowerScore Staff
  • PowerScore Staff
  • Posts: 5153
  • Joined: Apr 14, 2011
|
#76535
That is solid reasoning, sparrrkk_, and I had the same thought when I first read this question. To strengthen the argument we might have wanted to see an answer about the percentage of library users in the two communities being approximately the same, or perhaps higher in Glenwood. There is more than one way to strengthen, or weaken, an argument, so it's good to be prepared for several.
User avatar
 uvn5008
  • Posts: 1
  • Joined: Jul 22, 2021
|
#88960
I am still confused. The explanation says that we do not know the difference in size between Central Redville and Central Glenwood, but we do. The stimulus says that there are many more people in Central Glenwood. Can someone explain this?
 Robert Carroll
PowerScore Staff
  • PowerScore Staff
  • Posts: 1787
  • Joined: Dec 06, 2013
|
#88966
Hi uvn,

"Size" is ambiguous. And our initial post on this thread makes clear what we mean: area covered. We even talk about "walking distance". So the information we have about population tells us nothing about the area covered by the central regions of each city. That's the kind of size we'd also like to know about, and which answer choice (B) provides.

Robert Carroll
User avatar
 ashpine17
  • Posts: 321
  • Joined: Apr 06, 2021
|
#94891
People be debating the choices but I find the stimulus confusing. what exactly is the problem and how can you tell?
User avatar
 ashpine17
  • Posts: 321
  • Joined: Apr 06, 2021
|
#94892
is the argument assuming that all these people within walking distance of the library in Glenwood would actually use the library if it were put in their proximity? is that why a is incorrect? because the stimulus is talking about a future proposal while this choice is talking about how things currently are without the change?
 Adam Tyson
PowerScore Staff
  • PowerScore Staff
  • Posts: 5153
  • Joined: Apr 14, 2011
|
#94979
The argument concludes that the library will be close enough to walk for more users, ashpine, based on the fact that there are more people in the new location. There are a lot of problems with that, including the fact that we don't know if anyone from central Glenwood ever uses the library, or if they would start using it if it was closer to them. It's not necessary that everyone there would use it, but it is necessary that at least someone there would!

Answer A tells us nothing about what would happen if the library was in central Glenwood. It just says that it used to be somewhere else. Did more people use it when it was there? Did people from central Glenwood use it? Was it walkable for anyone? There's just so much we don't know, so it can't help or hurt the argument in any way.

Answer B isn't enough to prove the conclusion, because we still don't know if anyone in central Glenwood would use the library, but it helps because it means there are more people concentrated within the same size space. A higher population density would mean more people could be within walking distance. It's not a great and powerful strengthener, but it does go in the "plus" column for this author.

Get the most out of your LSAT Prep Plus subscription.

Analyze and track your performance with our Testing and Analytics Package.